theTeam-11 must win

This topic has expert replies
User avatar
GMAT Instructor
Posts: 3650
Joined: Wed Jan 21, 2009 4:27 am
Location: India
Thanked: 267 times
Followed by:80 members
GMAT Score:760

theTeam-11 must win

by sanju09 » Wed Nov 18, 2015 11:29 pm
The Team-11 has played half of its due number of matches so far, out of which it has won 40 percent of the matches and lost the rest of these. Which of the following value could represent the possible number of the remaining matches the Team-11 must win in order to register an overall win percent of 60? Consider no ties.
A. 35
B. 30
C. 27
D. 24
E. 18


[spoiler]Made Up![/spoiler]
Last edited by sanju09 on Thu Nov 19, 2015 4:56 am, edited 1 time in total.
The mind is everything. What you think you become. -Lord Buddha



Sanjeev K Saxena
Quantitative Instructor
The Princeton Review - Manya Abroad
Lucknow-226001

www.manyagroup.com

Master | Next Rank: 500 Posts
Posts: 363
Joined: Sun Oct 17, 2010 3:24 pm
Thanked: 115 times
Followed by:3 members

by theCEO » Thu Nov 19, 2015 1:09 am
sanju09 wrote:The Team-11 has played half of its due number of matches so far, out of which it has won 40 percent of the matches and lost the rest of these. Which of the following value could represent the possible number of the remaining matches the Team-11 must win in order to register an overall win percent of 60? Consider no ties.
A. 36
B. 30
C. 27
D. 24
E. 18


[spoiler]Made Up![/spoiler]
total games played = g
half of total games played = 0.5g

The number of games won in the first half + the number of games won in the second half = 60% of the total games played
0.4(0.5g) + x(0.5g) = 0.6g
0.2g + 0.5xg = 0.6g
0.5x = 0.4
x = 4/5 - > therefore team has to win 80% of the remaining games

4/5 * number of games in the second half = whole number
check answer choices for which choice is a multiple of 5

[spoiler]Only B meets the criteria; ans = b[/spoiler]



[spoiler][/spoiler]

User avatar
GMAT Instructor
Posts: 3650
Joined: Wed Jan 21, 2009 4:27 am
Location: India
Thanked: 267 times
Followed by:80 members
GMAT Score:760

by sanju09 » Thu Nov 19, 2015 5:08 am
theCEO wrote:
sanju09 wrote:The Team-11 has played half of its due number of matches so far, out of which it has won 40 percent of the matches and lost the rest of these. Which of the following value could represent the possible number of the remaining matches the Team-11 must win in order to register an overall win percent of 60? Consider no ties.
A. 35
B. 30
C. 27
D. 24
E. 18


[spoiler]Made Up![/spoiler]
total games played = g
half of total games played = 0.5g

The number of games won in the first half + the number of games won in the second half = 60% of the total games played
0.4(0.5g) + x(0.5g) = 0.6g
0.2g + 0.5xg = 0.6g
0.5x = 0.4
x = 4/5 - > therefore team has to win 80% of the remaining games

4/5 * number of games in the second half = whole number and the answer as well
check answer choices for which choice is a multiple of 5

[spoiler]Only B meets the criteria; ans = b[/spoiler]



[spoiler][/spoiler]
Hi theCEO,

There's a typo in option A, this is 35 not 36. Edited.

Your work is good, but in the end it didn't address the real question, which is "the possible number of the remaining matches the Team-11 must win". It's correct that the possible answer is 4/5 of some total and is a whole number too, and it's also true that each half of the total is a multiple of 5, but we should better look for a multiple of 4 to answer the exact question instead of a multiple of 5 here.
The mind is everything. What you think you become. -Lord Buddha



Sanjeev K Saxena
Quantitative Instructor
The Princeton Review - Manya Abroad
Lucknow-226001

www.manyagroup.com

Master | Next Rank: 500 Posts
Posts: 363
Joined: Sun Oct 17, 2010 3:24 pm
Thanked: 115 times
Followed by:3 members

by theCEO » Fri Nov 20, 2015 7:00 pm
sanju09 wrote:
theCEO wrote:
sanju09 wrote:The Team-11 has played half of its due number of matches so far, out of which it has won 40 percent of the matches and lost the rest of these. Which of the following value could represent the possible number of the remaining matches the Team-11 must win in order to register an overall win percent of 60? Consider no ties.
A. 35
B. 30
C. 27
D. 24
E. 18


[spoiler]Made Up![/spoiler]
total games played = g
half of total games played = 0.5g

The number of games won in the first half + the number of games won in the second half = 60% of the total games played
0.4(0.5g) + x(0.5g) = 0.6g
0.2g + 0.5xg = 0.6g
0.5x = 0.4
x = 4/5 - > therefore team has to win 80% of the remaining games

4/5 * number of games in the second half = whole number and the answer as well
check answer choices for which choice is a multiple of 5

[spoiler]Only B meets the criteria; ans = b[/spoiler]



[spoiler][/spoiler]
Hi theCEO,

There's a typo in option A, this is 35 not 36. Edited.

Your work is good, but in the end it didn't address the real question, which is "the possible number of the remaining matches the Team-11 must win". It's correct that the possible answer is 4/5 of some total and is a whole number too, and it's also true that each half of the total is a multiple of 5, but we should better look for a multiple of 4 to answer the exact question instead of a multiple of 5 here.
Hi sanju09,

I hate it when I don't answer questions properly! Thanks for the info. I will repost the full solution.

Master | Next Rank: 500 Posts
Posts: 363
Joined: Sun Oct 17, 2010 3:24 pm
Thanked: 115 times
Followed by:3 members

by theCEO » Fri Nov 20, 2015 7:12 pm
sanju09 wrote:The Team-11 has played half of its due number of matches so far, out of which it has won 40 percent of the matches and lost the rest of these. Which of the following value could represent the possible number of the remaining matches the Team-11 must win in order to register an overall win percent of 60? Consider no ties.
A. 35
B. 30
C. 27
D. 24
E. 18


[spoiler]Made Up![/spoiler]

total games played = g
half of total games played = (g/2) = 0.5g

The number of games won in the first half + the number of games won in the second half = 60% of the total games played
0.4(0.5g) + x(0.5g) = 0.6g
0.2g + 0.5xg = 0.6g
0.5x = 0.4
x = 4/5 - > therefore team has to win 80% of the remaining games

4/5 * number of games in the second half = whole number
4/5 * (g/2) = whole number; (g/2) also is a whole number
check each answer choice to see which produce a whole number when we have g/2

A. 35; 4/5 * (g/2) = 35; (g/2) = 43.75 -> not possible
B. 30; 4/5 * (g/2) = 30; (g/2) = 37.50 -> not possible
C. 27; 4/5 * (g/2) = 27; (g/2) = 33.75 -> not possible
D. 24; 4/5 * (g/2) = 24; (g/2) = 30 -> possible
E. 18; 4/5 * (g/2) = 18; (g/2) = 22.5 -> not possible

ans = d
Last edited by theCEO on Fri Nov 20, 2015 11:36 pm, edited 1 time in total.

Master | Next Rank: 500 Posts
Posts: 313
Joined: Tue Oct 13, 2015 7:01 am
Thanked: 2 times

by jain2016 » Fri Nov 20, 2015 11:31 pm
Hi All ,

Can you please advise alternate approach?

Thanks in advance.

Thanks,

SJ

User avatar
GMAT Instructor
Posts: 15539
Joined: Tue May 25, 2010 12:04 pm
Location: New York, NY
Thanked: 13060 times
Followed by:1906 members
GMAT Score:790

by GMATGuruNY » Sat Nov 21, 2015 3:58 am
sanju09 wrote:The Team-11 has played half of its due number of matches so far, out of which it has won 40 percent of the matches and lost the rest of these. Which of the following value could represent the possible number of the remaining matches the Team-11 must win in order to register an overall win percent of 60? Consider no ties.
A. 35
B. 30
C. 27
D. 24
E. 18
Let F = the first half of the season and S = the second half of the season.

Percentage attributed to F = 40%.
Percentage attributed to S = x%.
Percentage attributed to the MIXTURE of F and S = 60%.

Since F and S each consist of an EQUAL NUMBER OF GAMES, the percentage for the MIXTURE -- 60% -- must be HALFWAY between the percentages for F and S (40% and x%).
Thus:
x% = 80% = 4/5.

Since (4/5)S must be an INTEGER VALUE, S must be a MULTIPLE OF 5:
S=5 --> number of wins = (4/5)(5) = 4.
S=10 --> number of wins = (4/5)(10) = 8.
S=15 --> number of wins = (4/5)(15) = 12.
The values in red imply that the number of wins in S must be a MULTIPLE OF 4.

The correct answer is D.
Private tutor exclusively for the GMAT and GRE, with over 20 years of experience.
Followed here and elsewhere by over 1900 test-takers.
I have worked with students based in the US, Australia, Taiwan, China, Tajikistan, Kuwait, Saudi Arabia -- a long list of countries.
My students have been admitted to HBS, CBS, Tuck, Yale, Stern, Fuqua -- a long list of top programs.

As a tutor, I don't simply teach you how I would approach problems.
I unlock the best way for YOU to solve problems.

For more information, please email me (Mitch Hunt) at [email protected].
Student Review #1
Student Review #2
Student Review #3

User avatar
Legendary Member
Posts: 1100
Joined: Sat May 10, 2014 11:34 pm
Location: New Delhi, India
Thanked: 205 times
Followed by:24 members

by GMATinsight » Sat Nov 21, 2015 7:57 am
sanju09 wrote:The Team-11 has played half of its due number of matches so far, out of which it has won 40 percent of the matches and lost the rest of these. Which of the following value could represent the possible number of the remaining matches the Team-11 must win in order to register an overall win percent of 60? Consider no ties.
A. 35
B. 30
C. 27
D. 24
E. 18
[spoiler]Made Up![/spoiler]
Let, Matches played = 10
i.e. Total Matches = 2*10 = 20
Won = 4
Lost = 6
Minimum matches to win = x

Matches to be won = 12 out of 20
i.e. More matches to be won atleast = 12-4=8

i.e. Answer must be a multiple of 8

Answer: option D
"GMATinsight"Bhoopendra Singh & Sushma Jha
Most Comprehensive and Affordable Video Course 2000+ CONCEPT Videos and Video Solutions
Whatsapp/Mobile: +91-9999687183 l [email protected]
Contact for One-on-One FREE ONLINE DEMO Class Call/e-mail
Most Efficient and affordable One-On-One Private tutoring fee - US$40-50 per hour

Master | Next Rank: 500 Posts
Posts: 274
Joined: Fri Sep 18, 2015 10:58 pm
Thanked: 12 times
Followed by:1 members
GMAT Score:530

by vishalwin » Sun Nov 22, 2015 12:23 pm
Hi Mitch,

I did this question by 4 variable cross table method and I got answer = 40




|Played | Left |
-----|-----------|---------|-------
Won| 20 | M |60
------|-----------|---------|-------
Lost| 30 | |
-----|-----------|---------|-------
| 50 | 50 |100






From above value of M must be 40 to make it 60 won out of hundred.


Can you please tell me where I am making mistake.
-
Thanks & Regards
vishalwin
------------------------------------
GMAT Score - 530
I will BEAT the GMAT!

User avatar
GMAT Instructor
Posts: 15539
Joined: Tue May 25, 2010 12:04 pm
Location: New York, NY
Thanked: 13060 times
Followed by:1906 members
GMAT Score:790

by GMATGuruNY » Mon Nov 23, 2015 3:07 am
vishalwin wrote:From above value of M must be 40 to make it 60 won out of hundred.
Can you please tell me where I am making mistake.
The question stems asks what COULD be the number of remaining games that are won.
In your solution, the total number of games = 100, with the result that the number of remaining games that are won = 40.
Implication:
The number of remaining games that are won COULD be 40.
But 40 is not among the answer choices.

To determine which of the five answer choices could be the number of remaining games that are won, test the LEAST POSSIBLE CASE.
Since 2/5 of the played games have been won, the least possible value for the number of played games = 5.
The following matrix is yielded:
Image

Since 40% of the played games have been won, the number of played games that have been won = 40% of 5 = 2.
Since 60% of the total number of games are won, the total number of games that are won = 60% of 10 = 6.
The following matrix is yielded:
Image
In the resulting matrix, the number of remaining games that are won = 4.

If the total number of games is multiplied by a factor of x, then the number of remaining games that are won will also be multiplied by a factor of x.
Implication:
The number of remaining games that are won = MULTIPLE OF 4.
Of the five answer choices, only D is a multiple of 4.
Private tutor exclusively for the GMAT and GRE, with over 20 years of experience.
Followed here and elsewhere by over 1900 test-takers.
I have worked with students based in the US, Australia, Taiwan, China, Tajikistan, Kuwait, Saudi Arabia -- a long list of countries.
My students have been admitted to HBS, CBS, Tuck, Yale, Stern, Fuqua -- a long list of top programs.

As a tutor, I don't simply teach you how I would approach problems.
I unlock the best way for YOU to solve problems.

For more information, please email me (Mitch Hunt) at [email protected].
Student Review #1
Student Review #2
Student Review #3

GMAT/MBA Expert

User avatar
GMAT Instructor
Posts: 16207
Joined: Mon Dec 08, 2008 6:26 pm
Location: Vancouver, BC
Thanked: 5254 times
Followed by:1268 members
GMAT Score:770

by Brent@GMATPrepNow » Mon Nov 23, 2015 9:42 am
I thought I'd point out that Mitch's approach (aka Double Matrix Method) can be used for most questions featuring a population in which each member has two characteristics associated with it.
Here, we have a population of matches, and the two characteristics are:
- win or loss
- already played or remaining

This question type is VERY COMMON on the GMAT, so be sure to master the technique.

To learn more about the Double Matrix Method, watch our free video: https://www.gmatprepnow.com/module/gmat- ... ems?id=919

Once you're familiar with this technique, you can attempt these additional practice questions:

Easy Problem Solving questions
- https://www.beatthegmat.com/the-aam-aadm ... 72242.html
- https://www.beatthegmat.com/finance-majo ... 67425.html

Medium Problem Solving questions
- https://www.gmatprepnow.com/module/gmat- ... /video/920
- https://www.beatthegmat.com/probability- ... 73360.html
- https://www.beatthegmat.com/posted-speed ... 72374.html
- https://www.beatthegmat.com/motel-t271938.html
- https://www.beatthegmat.com/of-the-appli ... 70255.html
- https://www.beatthegmat.com/opening-nigh ... 64869.html
- https://www.beatthegmat.com/at-least-100 ... 74669.html
- https://www.beatthegmat.com/prblem-solving-t279424.html

Difficult Problem Solving questions
- https://www.gmatprepnow.com/module/gmat- ... /video/946
- https://www.beatthegmat.com/ratio-problem-t268339.html
- https://www.beatthegmat.com/overlapping- ... 65223.html
- https://www.beatthegmat.com/fractions-t264254.html
- https://www.beatthegmat.com/overlapping- ... 64092.html
- https://www.beatthegmat.com/mba/2011/05/ ... question-2

Easy Data Sufficiency questions
- https://www.gmatprepnow.com/module/gmat- ... /video/943
- https://www.beatthegmat.com/for-what-per ... 70596.html
- https://www.beatthegmat.com/ds-quest-t187706.html

Medium Data Sufficiency questions
- https://www.beatthegmat.com/sets-matrix-ds-t271914.html
- https://www.beatthegmat.com/each-of-peop ... 71375.html
- https://www.beatthegmat.com/a-manufacturer-t270331.html
- https://www.beatthegmat.com/in-costume-f ... 69355.html
- https://www.beatthegmat.com/mba/2011/05/ ... question-1

Difficult Data Sufficiency questions
- https://youtu.be/dsCeqF9Kbk8
- https://www.beatthegmat.com/double-set-m ... 71423.html
- https://youtu.be/dOZ9KM1m5Hs
- https://www.beatthegmat.com/sets-t269449.html
- https://www.beatthegmat.com/mba/2011/05/ ... question-3

Cheers,
Brent
Brent Hanneson - Creator of GMATPrepNow.com
Image

Master | Next Rank: 500 Posts
Posts: 274
Joined: Fri Sep 18, 2015 10:58 pm
Thanked: 12 times
Followed by:1 members
GMAT Score:530

by vishalwin » Tue Nov 24, 2015 1:11 am
Hi Mitch,

How to choose if we have any other option like 36?

In the question it's not written to find out least possible but rather must be won.


Can you please help.
Thanks & Regards
vishalwin
------------------------------------
GMAT Score - 530
I will BEAT the GMAT!

User avatar
GMAT Instructor
Posts: 15539
Joined: Tue May 25, 2010 12:04 pm
Location: New York, NY
Thanked: 13060 times
Followed by:1906 members
GMAT Score:790

by GMATGuruNY » Tue Nov 24, 2015 5:05 am
vishalwin wrote:Hi Mitch,

How to choose if we have any other option like 36?

In the question it's not written to find out least possible but rather must be won.


Can you please help.
The answer choices would not include both 24 and 36, since either is a viable option for the number of remaining games that are won.
Question stem: which of the following values could...
When the question stem includes the words in red, only ONE answer choice will be viable.
Here -- since the number of remaining games that are won must be a multiple of 4 -- the only viable answer choice is D.
Private tutor exclusively for the GMAT and GRE, with over 20 years of experience.
Followed here and elsewhere by over 1900 test-takers.
I have worked with students based in the US, Australia, Taiwan, China, Tajikistan, Kuwait, Saudi Arabia -- a long list of countries.
My students have been admitted to HBS, CBS, Tuck, Yale, Stern, Fuqua -- a long list of top programs.

As a tutor, I don't simply teach you how I would approach problems.
I unlock the best way for YOU to solve problems.

For more information, please email me (Mitch Hunt) at [email protected].
Student Review #1
Student Review #2
Student Review #3

GMAT Instructor
Posts: 2630
Joined: Wed Sep 12, 2012 3:32 pm
Location: East Bay all the way
Thanked: 625 times
Followed by:119 members
GMAT Score:780

by Matt@VeritasPrep » Fri Nov 27, 2015 2:01 am
The matrix method works, but I'd go for an equation instead, something like

(40% of the first half) + (x% of the second half) = 60% of the total

or

.4(m/2) + x(m/2) = .6m

so

x = .8

and the team must win 80% of its remaining matches.

Since the answer must be 80% of an integer, look for an answer that divides by 4. (80% = 4/5, so our number is a multiple of 4 and the total number of matches is a multiple of 5.) 24 is the only candidate, so we're done.